LSAT and Law School Admissions Forum

Get expert LSAT preparation and law school admissions advice from PowerScore Test Preparation.

 Administrator
PowerScore Staff
  • PowerScore Staff
  • Posts: 8916
  • Joined: Feb 02, 2011
|
#82068
Complete Question Explanation

The correct answer choice is (D).

Answer choice (A):

Answer choice (B):

Answer choice (C):

Answer choice (D): This is the correct answer choice.

Answer choice (E):


This explanation is still in progress. Please post any questions below!
 emilysnoddon
  • Posts: 64
  • Joined: Apr 22, 2016
|
#26055
I had difficulty with this question. I chose E because I realized this was the argument that Garcia was making for which the author says is flawed. I recognize that he does give Garcia some credit in the first paragraph so can understand why this answer is incorrect but I was wonder where the support is for answer choice D. Is it in the last paragraph?
 Ladan Soleimani
PowerScore Staff
  • PowerScore Staff
  • Posts: 43
  • Joined: Oct 08, 2015
|
#26126
Hey Emily,

The author has two main criticisms of Garcia's work. The first is that Garcia's evidence is inconsistent. That is what answer choice (E) is referring to; however, the answer doesn't actually address the issues that author points outs. The author is not uncertain about the relationships between the League of United Latin American Citizens and the Congress of Spanish-Speaking People, but thinks Garcia is washing over some of their beliefs. Garcia claims those two groups had underlying consensus on radical issues, but the author points out all the opposing viewpoints that the group held which would undermine Garcia's position.

The support for answer choice (D) is in the last paragraph. The author's second criticism is that the views of these activists may not represent the views of the greater Mexican American population (line 36).

Ladan
 Etsevdos
  • Posts: 62
  • Joined: Oct 22, 2017
|
#41750
How about A. Is it bc number vs percent and also bc "one cannot assume"(line 57)? My issue with second point is that "cannot assume" suggests uncertain, not necessarily false. What am I missing on this point?
 nicholaspavic
PowerScore Staff
  • PowerScore Staff
  • Posts: 271
  • Joined: Jun 12, 2017
|
#42340
Hi etsevdos,

If I can jump in for Ladan here, what's important to understand about this passage for this question is the lack of certainty about the relationship and its consistency of views between the two groups discussed in the lines cited by Ladan.

And the deal with (A) is that the LSAT is trying to game you here. Don't fall for it. Of course, if you read the lines you cite in isolation from the rest of the passage, you can easily walk away with the mistaken impression that the author is uncertain about whether Mexican Americans born here resulted in more activism, but that's not what she's saying at all. She's using that example to attack the ideas that Garcia made. She's not saying that "one cannot assume," she's actually saying Garcia cannot assume. That's the concept which you are supposed to take from the closing lines. So she's actually not expressing uncertainty at all. She saying Garcia is full of it in that example.

The takeaway here is don't read sentences in isolation. Always make connections between what you are reading and what you just read.

Thanks for the great question and I hope this helps! :-D

Get the most out of your LSAT Prep Plus subscription.

Analyze and track your performance with our Testing and Analytics Package.